Bài tập  /  Bài đang cần trả lời

So sánh các số sau

so sánh các số sau
a)72^45-72^44 và 72^44-72^43
b)2^500 và 5^200
c)31^11 và 17^14
d)3^24680 và 2^37020
e)2^1050 và 5^400
g)5^2n và 2^5n (n thuộc N)
 
2 trả lời
Hỏi chi tiết
34
Tham khảo câu trả lời từ Trợ lý ảo Lazi
Để so sánh các số trong các bài toán này, chúng ta có thể sử dụng một số phương pháp như so sánh trực tiếp, sử dụng logarithm, hoặc phân tích các biểu thức.

a) So sánh \(72^{45} - 72^{44}\) và \(72^{44} - 72^{43}\):

Ta có:
\[ 72^{45} - 72^{44} = 72^{44}(72 - 1) = 72^{44} \cdot 71 \]
\[ 72^{44} - 72^{43} = 72^{43}(72 - 1) = 72^{43} \cdot 71 \]

Rõ ràng \(72^{44} \cdot 71\) lớn hơn \(72^{43} \cdot 71\) vì \(72^{44} > 72^{43}\).

Vậy:
\[ 72^{45} - 72^{44} > 72^{44} - 72^{43} \]

b) So sánh \(2^{500}\) và \(5^{200}\):

Ta có thể sử dụng logarithm để so sánh:
\[ \log(2^{500}) = 500 \log 2 \]
\[ \log(5^{200}) = 200 \log 5 \]

So sánh \(500 \log 2\) và \(200 \log 5\):
\[ \frac{500 \log 2}{200 \log 5} = \frac{5 \log 2}{2 \log 5} \]

Giá trị của \(\log 2 \approx 0.3010\) và \(\log 5 \approx 0.6990\):
\[ \frac{5 \cdot 0.3010}{2 \cdot 0.6990} \approx \frac{1.505}{1.398} \approx 1.077 \]

Vì \(1.077 > 1\), nên:
\[ 2^{500} > 5^{200} \]

c) So sánh \(31^{11}\) và \(17^{14}\):

Ta có thể sử dụng logarithm để so sánh:
\[ \log(31^{11}) = 11 \log 31 \]
\[ \log(17^{14}) = 14 \log 17 \]

So sánh \(11 \log 31\) và \(14 \log 17\):
\[ \frac{11 \log 31}{14 \log 17} \]

Giá trị của \(\log 31 \approx 1.491\) và \(\log 17 \approx 1.230\):
\[ \frac{11 \cdot 1.491}{14 \cdot 1.230} \approx \frac{16.401}{17.22} \approx 0.953 \]

Vì \(0.953 < 1\), nên:
\[ 31^{11} < 17^{14} \]

d) So sánh \(3^{24680}\) và \(2^{37020}\):

Ta có thể sử dụng logarithm để so sánh:
\[ \log(3^{24680}) = 24680 \log 3 \]
\[ \log(2^{37020}) = 37020 \log 2 \]

So sánh \(24680 \log 3\) và \(37020 \log 2\):
\[ \frac{24680 \log 3}{37020 \log 2} \]

Giá trị của \(\log 3 \approx 0.4771\) và \(\log 2 \approx 0.3010\):
\[ \frac{24680 \cdot 0.4771}{37020 \cdot 0.3010} \approx \frac{11774.428}{11141.02} \approx 1.057 \]

Vì \(1.057 > 1\), nên:
\[ 3^{24680} > 2^{37020} \]

e) So sánh \(2^{1050}\) và \(5^{400}\):

Ta có thể sử dụng logarithm để so sánh:
\[ \log(2^{1050}) = 1050 \log 2 \]
\[ \log(5^{400}) = 400 \log 5 \]

So sánh \(1050 \log 2\) và \(400 \log 5\):
\[ \frac{1050 \log 2}{400 \log 5} \]

Giá trị của \(\log 2 \approx 0.3010\) và \(\log 5 \approx 0.6990\):
\[ \frac{1050 \cdot 0.3010}{400 \cdot 0.6990} \approx \frac{316.05}{279.6} \approx 1.13 \]

Vì \(1.13 > 1\), nên:
\[ 2^{1050} > 5^{400} \]

g) So sánh \(5^{2n}\) và \(2^{5n}\) (với \(n \in \mathbb{N}\)):

Ta có thể sử dụng logarithm để so sánh:
\[ \log(5^{2n}) = 2n \log 5 \]
\[ \log(2^{5n}) = 5n \log 2 \]

So sánh \(2n \log 5\) và \(5n \log 2\):
\[ \frac{2n \log 5}{5n \log 2} = \frac{2 \log 5}{5 \log 2} \]

Giá trị của \(\log 5 \approx 0.6990\) và \(\log 2 \approx 0.3010\):
\[ \frac{2 \cdot 0.6990}{5 \cdot 0.3010} \approx \frac{1.398}{1.505} \approx 0.929 \]

Vì \(0.929 < 1\), nên:
\[ 5^{2n} < 2^{5n} \]
1
0
Linh say bye
21/07 21:27:58
+5đ tặng

Mở khóa để xem toàn bộ nội dung trả lời

(?)
Bạn đã đạt đến giới hạn của mình. Bằng cách Đăng ký tài khoản, bạn có thể xem toàn bộ nội dung trả lời
Cải thiện điểm số của bạn bằng cách đăng ký tài khoản Lazi.
Xem toàn bộ các câu trả lời, chat trực tiếp 1:1 với đội ngũ Gia sư Lazi bằng cách Đăng ký tài khoản ngay bây giờ
Tôi đã có tài khoản? Đăng nhập
2
0
Trúc Mai
21/07 21:28:15
+4đ tặng

Bạn hỏi - Lazi trả lời

Bạn muốn biết điều gì?

GỬI CÂU HỎI
Học tập không giới hạn cùng học sinh cả nước và AI, sôi động, tích cực, trải nghiệm
Câu hỏi Toán học Lớp 6 mới nhất

Hôm nay bạn thế nào? Hãy nhấp vào một lựa chọn, nếu may mắn bạn sẽ được tặng 50.000 xu từ Lazi

Vui Buồn Bình thường

Học ngoại ngữ với Flashcard

×
Gia sư Lazi Gia sư
×
Trợ lý ảo Trợ lý ảo